RegistrierenRegistrieren   LoginLogin   FAQFAQ    SuchenSuchen   
Sommerfeld-Wilson-Quantisierungsbedingung.
 
Neue Frage »
Antworten »
    Foren-Übersicht -> Quantenphysik
Autor Nachricht
kreis



Anmeldungsdatum: 27.04.2014
Beiträge: 93

Beitrag kreis Verfasst am: 07. Nov 2014 02:04    Titel: Sommerfeld-Wilson-Quantisierungsbedingung. Antworten mit Zitat

Ich lese gerade über das Bohr-Sommerfeld-Modell nach und verstehe alles außer diesen Zwischenschritt:



Also in der Beispielaufgabe geht man von einem Körper aus, der sich frei um eine fixierte Achse bewegen kann und die Begründung soll deswegen folgende sein:

"For a body rotating freely about a fixed axis, the
angular momentum (L) will be canonically conjugate to the angular displacement (θ). Over one complete period, θ varies from 0 to 2π helping us in fixing the limits of integration. Therefore,...." (Siehe Seite 42-43 http://physlab.lums.edu.pk/images/7/7a/Modernphysics2009.pdf )

Meine Idee:
Hat das Irgendetwas mit dem Drehimpulserhaltungssatz zu tun? Dann könnte man nämlich vielleicht davon ausgehen dass dL/dt=0 bzw. d(mvr)/dt =d(pr)/dt =0 und sich ab da irgendwie weiter durchbeissen bis man die Umformung geschafft hat. Big Laugh

PS: Nur noch so ein Gedanke. Kann es sein dass ich evtl. in ein Polarkoordinatensystem wechseln muss? grübelnd
TomS
Moderator


Anmeldungsdatum: 20.03.2009
Beiträge: 17896

Beitrag TomS Verfasst am: 07. Nov 2014 06:47    Titel: Antworten mit Zitat

Das erste Kurvenintegral über dq ist im klassischen Phasenraum formuliert. q ist ein "verallgemeinerter" Ort, p der zu q kanonisch konjugierte "verallgemeinerte" Impuls. Diese. Koordinatenwahl im Phasenraum ist recht beliebig und kann dem jeweiligen Problem angepasst werden.

http://de.wikipedia.org/wiki/Phasenraum

Das zweite Kurvenintegral ist nun für zwei sehr spezielle Koordinaten formuliert. Und zwar betrachtet man speziell für ein rotationsinvariantes System als Koordinate den Winkel theta sowie den zugehörigen kanonisch konjugierten Impuls, den Drehimpuls L. Das Integral über eine geschlossene Kurve im Phasenraum entspricht nun tatsächlich der Integration von Null bis 2*pi über den Winkel theta.

Und damit liegt tatsächlich soetwas wie ein Polarkoordinatensystem im Ortsraum (theta,r) vor; im Phasenraum mit den Variablen (q,p) bzw. jetzt (theta,L) sind das aber keine Polarkoordinaten.

Mit dem Drehimpulserhaltungssatz hat das insoferen etwas zu tun, als man die speziellen Koordinaten so wählen kann, dass der kanonisch konjugiere Impuls (hier L) in diesem speziellen Fall erhalten ist. Deswegen ist gerade diese Koordinatenwahl dem Problem am besten angelasst. L ist entlang der Kurve, die das System im Phasenraum durchläuft, konstant.

Schreiben wir ausführlich



In diesem speziellen Fall ist L entlang der physikalisch realisierten Kurve konstant; und genau entlang so einer Kurve soll gemäß der Quantisierungsbedingung integriert werden.

(ich würde mir dazu aber nicht zu viele Gedanken machen, weil das nur für sehr wenige klassische Systeme zu vernünftigen Quantisierungsbedingungen führt)

_________________
Niels Bohr brainwashed a whole generation of theorists into thinking that the job (interpreting quantum theory) was done 50 years ago.
kreis



Anmeldungsdatum: 27.04.2014
Beiträge: 93

Beitrag kreis Verfasst am: 07. Nov 2014 09:50    Titel: Antworten mit Zitat

Dankesehr! Sehr gute Erklärung! smile

Trotzdem noch eine kleine Frage:

Also unter einem frei rotierenden Körper habe ich mir z.B einen unendlich kleinen Massepunkt vorgestellt der sich auf irgendeiner Bahn periodisch um eine definierte Achse bewegt...

Also wäre sowohl der minimale Abstand von der Rotationsachse bis zum Punkt, als auch die Geschwindigkeit variabel.

Nichtsdestotrotz wird ja hier dann ausgesagt dass man nur die Änderung eines Winkels betrachtet, woraus folgt, dass der Radius konstant bleibt (sprich es handelt sich dann um einen Kreis).

Könnte sich, in diesem Fall ein Elektron, nicht auch elipsenförmig oder in irgendeiner unregelmäßigen Form um einen wohldefinierten statischebn Punkt (in guter Näherung der Kern) bewegen?..

Ich glaube ich verstehe das Konzept des ""frei" rotierenden Körpes um eine Achse" falsch... kann das sein? xD
TomS
Moderator


Anmeldungsdatum: 20.03.2009
Beiträge: 17896

Beitrag TomS Verfasst am: 07. Nov 2014 13:14    Titel: Antworten mit Zitat

kreis hat Folgendes geschrieben:
Dankesehr! Sehr gute Erklärung! :)

Trotzdem noch eine kleine Frage:

Also unter einem frei rotierenden Körper habe ich mir z.B einen unendlich kleinen Massepunkt vorgestellt der sich auf irgendeiner Bahn periodisch um eine definierte Achse bewegt...

kann man sich zunächst so vorstellen, ja

kreis hat Folgendes geschrieben:
Nichtsdestotrotz wird ja hier dann ausgesagt dass man nur die Änderung eines Winkels betrachtet, woraus folgt, dass der Radius konstant bleibt (sprich es handelt sich dann um einen Kreis).

Nein, das wird gerade nicht ausgesagt, weil die Integrale nicht im Konfigurationsraum (Ortsraum) sondern im Phasenraum definiert sind. Im hier betrachteten Spezialfall ist die "radiale" Koordinate, die entlang des Integrationsweges konstant bleibt, gerade der Drehimpuls L.

http://en.wikipedia.org/wiki/Hamiltonian_mechanics
http://en.wikipedia.org/wiki/Canonical_coordinates
http://en.wikipedia.org/wiki/Action-angle_coordinates

kreis hat Folgendes geschrieben:
Könnte sich, in diesem Fall ein Elektron, nicht auch ellipsenförmig oder in irgendeiner unregelmäßigen Form um einen wohldefinierten statischen Punkt (in guter Näherung der Kern) bewegen?..

Ja, das Elektron kann sich im Ortsraum durchaus auf einer Ellipse bewegen. Aber das Integral erfolgt nicht im Ortsraum (r, theta) entlang der Ellipse sondern im Phasenraum (theta, L).

_________________
Niels Bohr brainwashed a whole generation of theorists into thinking that the job (interpreting quantum theory) was done 50 years ago.
TomS
Moderator


Anmeldungsdatum: 20.03.2009
Beiträge: 17896

Beitrag TomS Verfasst am: 08. Nov 2014 08:24    Titel: Antworten mit Zitat

Ein kurzes Beispiel, um den Unterschied zu erklären. Betrachte ein Pendel mit



Der Impuls lautet



Dies ist die Parameterdarstellung der Bewegung im zweidimensionalen Phasenraum (q,p). Sie entspricht offensichtlich einer Ellipse. Im Ortsraum liegt eine eindimensionale, periodische Bewegung in q(t) vor, und das ist sicher keine Ellipse (q entspricht hier auch im Ortsraum einem Winkel).

Betrachten wir nun ein Pendel mit Überschlag. Dafür gelten die kleinen Schwingungen, d.h. die o.g. Gleichungen mit sin und cos nicht mehr. Es liegt auch keine Ellipse im Phasenraum vor, sondern eine Art wellenförmige Bewegung; insbs. ist der Winkel zwischen q und p im Phasenraum unbeschränkt! Im Ortsraum liegt dagegen jetzt ein sich überschlagendes Pendel vor, das selbstverständlich eine Kreisbewegung (mit variabler Winkelgeschwindigkeit) durchführt.

Die Bahnen im Ortsraum und im Phasenraum sind also etwas grundverschiedenes.

Das Integral über dq im Phasenraum wird über eine volle Schwingungsperiode ausgeführt. Natürlich entspricht dies bei einer periodischen Bewegung im Ortsraum ebenfalls einer Periode im Ortsraum. Aber die Integrationsvariablen sind andere. Beim Pendel sieht man das sehr schön, da man zwar anschaulich eine Pendelbewegung in einer zweidimensionalen Ebene vor sich sieht, mathematisch jedoch nur ein einziger Freiheitsgrad, nämlich der Winkel vorliegt.

_________________
Niels Bohr brainwashed a whole generation of theorists into thinking that the job (interpreting quantum theory) was done 50 years ago.
Neue Frage »
Antworten »
    Foren-Übersicht -> Quantenphysik